[obm-l] Dúvida ( área mínima )

2007-05-15 Thread cleber vieira
Amigos gostaria da ajuda de vocês neste seguinte problema. Duas retas r e s são parelelas, os pontos B de r e C de s são móveis e o ponto A, entre as retas é fixo; dentre todos os triângulos ABC, retângulos em A, teremos o de área mínima quando? a) teta = pi/3 b) teta = pi/4 obs: t

Re: [obm-l] dúvida ( cálculo)

2007-05-16 Thread cleber vieira
Valeu Bruno Muito obrigado Vieira __ Fale com seus amigos de graça com o novo Yahoo! Messenger http://br.messenger.yahoo.com/

Re: [obm-l] Congruência - Dúvida

2007-05-20 Thread Marcelo Salhab Brogliato
Olá, vamos tentar provar o seguinte teorema: Seja p um numero primo, entao: a = +- 1 (mod p) sss a^2 = 1 (mod p) ida: trivial.. volta: a^2 - 1 = 0 (mod p) (a+1)(a-1) = 0 (mod p) assim, p divide (a+1) ou (a-1).. logo: a+1 = 0 (mod p) ... a = -1 (mod p) ou: a-1 = 0 (mod p) ... a = 1 (mod p)

[obm-l] dúvida sobre Limite

2007-06-28 Thread Kleber Bastos
Como eu faço para provar a seguinte afirmativa : lim e^(x) = 1 , quando x tende para zero .

Re: [obm-l] Dúvida Continuidade

2007-07-11 Thread Luis Matos
Supondo que f e continua na origem, deve existir um d(elta) > 0 tal que para todo x satisfazendo |x| < d entao |f(x) - f(0)| < eps (para algum eps > 0). Mas como f(0) = 0 (basta fazer x = x + 0 e utilizar a propriedade) temos |f(x)| < eps para todo x com |x| 0, entao, para uma vizinhança de x0 de

Re: [obm-l] Dúvida Continuidade

2007-07-11 Thread Marcelo Salhab Brogliato
Olá Kleber, vamos dar uns chutes para x e y e encontrar umas propriedades dessas funcoes: y=0... f(x+0) = f(x) + f(0) f(0) = 0 x=-y... f(x-x) = f(x) + f(-x) f(-x) = -f(x) [funcao impar] x=y... f(x+x) = f(x) + f(x) f(2x) = 2f(x) [por inducao, facilmente mostramos que f(nx) = nf(x) p

[obm-l] Dúvida na interpretação

2007-08-06 Thread Igor Battazza
Comecei a estudar um livro sobre Teoria dos Números e logo no inicio o autor faz a seguinte definição: "Se a e b são inteiros dizemos que a divide b, denotado por a|b, se existir um inteiro c tal que b = a*c." Em seguida há um teorema que na verdade são as propriedades da divisão. "A divisão tem

Re: [obm-l] Primeira dúvida

2007-11-12 Thread ralonso
. Sérgio Martins da Silva wrote: > Caros participantes da lista, Gosto de matemática e estou chegando > agora à lista. Eis minha primeira dúvida: Quanto é i ^ i ? Significa > alguma coisa? > > Sérgio

Re: [obm-l] Primeira dúvida

2007-11-12 Thread Maurício Collares
Como i = e^(i*pi/2), temos que i^i é igual a (e^(i*pi/2))^i = e^(-1*pi/2) = 1/e^(pi/2) = 0,207879576. -- Abraços, Maurício On Nov 11, 2007 10:48 PM, Sérgio Martins da Silva <[EMAIL PROTECTED]> wrote: = Instruções para entrar

Re: [obm-l] Primeira dúvida

2007-11-12 Thread ralonso
verdade. Na mensagem anterior há um erro: i^i = cosh pi - sinh pi Maurício Collares wrote: > Como i = e^(i*pi/2), temos que i^i é igual a (e^(i*pi/2))^i = > e^(-1*pi/2) = 1/e^(pi/2) = 0,207879576. > > -- > Abraços, > Maurício > > On Nov 11, 2007 10:48 PM, Sérgio Martins da Silva <[EMAIL PROTECTED

Re: [obm-l] Primeira dúvida

2007-11-12 Thread Angelo Schranko
creveu: Caros participantes da lista, Gosto de matemática e estou chegando agora à lista. Eis minha primeira dúvida: Quanto é i ^ i ? Significa alguma coisa? Sérgio - Abra sua conta no Yahoo! Mail, o único sem limite de espaç

[obm-l] Dúvida em Geometria

2011-05-28 Thread Paulo Barclay Ribeiro
prezados,   Desculpe a dúvida, mas estou encontrando dificuldade num problema bem elementar, e peço uma orientação , é o seguinte:   Qual o cosseno do ângulo diedro entre duas faces de uma piramide quadrangular de altura dois e aresta da base igual a 8..Desde já agradeço.   Paulo

[obm-l] dúvida sobre séries

2011-06-06 Thread claudinei
Pessoal definitivamente nao consigo entender pq a série [somatória de 1/n] com n indo de 1 ao infinito, divergepq nao converge para zero alguém por favor poderia me explicar???

[obm-l] Dúvida em gabarito

2011-11-26 Thread marcone augusto araújo borges
seja um quadrado ABCD de lado a.Os arcos AC e BD formam com 3 dos lados do quadrado dois setores ciculares(um quarto do círculo de raio a cada).Deteminar a área comum aos setores circulares. A resposta que o livro dá é a^2(9raiz(3) - 4pi)/12 e a que eu achei foi a^2(4pi - 3raiz(3))/12 Dsculpem

[obm-l] Dúvida de Lógica

2012-02-07 Thread Pedro Chaves
Caros Colegas, Pode-se dizer que "2 > 3 => Todo brasileiro é desonesto"?   (O símbolo => indica implicação lógica.) Sei que é verdadeira a proposição condicional "Se 2 > 3, então todo brasileiro é desonesto", mas me parece que não existe implicação lógica. Desde já, muito obrigado. Um abraço

[obm-l] equação diferencial dúvida

2013-06-19 Thread Hermann
Meus amigos, agradeço a ajuda: Considere a eq dif y' = (2x + x.cos(x))/2y y = x + x.cos(x)/2 é uma solução para esta eq dif? Cheguei na seguinte equação cos(x)-x.sen(x)=2 e travei. Agradeço Hermann -- Esta mensagem foi verificada pelo sistema de antivírus e acredita-se estar livre de perigo.

[obm-l] Dúvida(questão simples)

2014-02-21 Thread marcone augusto araújo borges
Exatamente no momento em que o ponteiro das horas passa pelo 12, uma formiga começa a andar ao longo da borda de um relógio no sentido anti-horário,partindo do 6,com velocidade constante.Quando a formiga en- contra o ponteiro das horas,ela muda de direção e continua a andar na mesma velocidade

Re: [obm-l] Dúvida simples!!

2014-09-01 Thread Artur Costa Steiner
Se o preço da entrada for p>= 10, então teremos que P = 1000 - (p - 10)/2 . 40 = 1000 - 20p + 200 = 1200 - 20p Logo, p = (1200 - P)/20 = 60 - P/20, 0 <= P <= 1000 E o faturamento é F = p P = 60P - (P^2)/20, 0 <= P <= 1000 Artur Costa Steiner > Em 01/09/2014, às 22:35, Cláudio Thor escreveu:

[obm-l] Dúvida sobre Álgebra

2015-04-06 Thread Listeiro 037
Saudações. Tenho a dúvida sobre como se pode demonstrar (se for realmente verdade) que se 'p' é primo e divide uma circunferência com instrumentos euclidianos, então p-1 e p-2 também a divide. Ou seja, se existirem infinitos pp então existem infinitas tríades de consecutivos. Na ver

Re: [obm-l] Dúvida Análise

2015-05-05 Thread Artur Costa Steiner
Vc está se referindo a séries de funções? Se estiver, a resposta de modo geral é não. O que garante a igualdade é convergência uniforme. Artur Costa Steiner > Em 05/05/2015, às 03:43, Israel Meireles Chrisostomo > escreveu: > > Em toda série convergente o limite da soma é a soma dos limtes

[obm-l] Dúvida Análise:séries formais

2015-05-06 Thread Israel Meireles Chrisostomo
Se eu tiver uma série dupla do tipo ΣΣa_k(n), então, eu posso aplicar a ideia de convergência uniforme para dizer que o limite da soma é a soma dos limites, isto é, passar o limite para dentro do somatório?No caso, eu poderia dizer que se |a_k(n)|inf ΣΣa_k(n)=inf ΣΣ lim n-> a_k(n)?Eu tennho uma int

[obm-l] Re: Dúvida Desigualdades

2015-06-16 Thread Israel Meireles Chrisostomo
Considere reais positivos, por exemplo, uma ideia de provar isso seria ver se a,b e c são números então ab+bc+ac=K para um constante K qualquer, observando que a substituição implica que ab=z², ac=y² e bc=x², teremos que ter x²+y²+z²=K, ora mas para qualquer real existem x,y e z que satisfazem x²+y

[obm-l] Re: Dúvida Desigualdades

2015-06-16 Thread Israel Meireles Chrisostomo
Eu poderia supor o contrário, isto é, supor que não existem reais que satisfazem o enunciado e então chegar ao absurdo, pois isto implica que não existem reais tais que x²+y²+z²=K, o que é falso, pois sempre existem tais reais Em 16 de junho de 2015 20:10, Israel Meireles Chrisostomo < israelmchri

[obm-l] Dúvida sistemas lineares

2015-07-08 Thread Israel Meireles Chrisostomo
Dados a,b e c reais quaisquer sempre existem x,y e z tais que a=y+z, b=x+z e c=x+y? -- Esta mensagem foi verificada pelo sistema de antiv�rus e acredita-se estar livre de perigo.

[obm-l] Dúvida "aparentemente" simples

2015-08-11 Thread Vanderlei Nemitz
Trabalho com edição de material didático e outro dia, em conversa com um autor de material do ensino médio, surgiu uma questão sobre a qual gostaria de saber a opinião de vocês. O oposto de zero é zero ou não faz sentido falar dele? Por um lado, a solução da equação x = -x é x = 0, ou seja, poder

[obm-l] Re: Indução dúvida

2016-01-18 Thread Israel Meireles Chrisostomo
Por exemplo, eu quero provar que f(n)>c para todo n inteiro.Então, eu provei o caso base,e considerei a hipótese de indução, suponha que é válido para um k que f(k)>c e supus que f(k+1)c e f(k+1)c, o que é uma contradição, pois f(k+1) não pode ser maior e menor do que c ao mesmo tempo. Então, isto

[obm-l] Dúvida em Logaritmos

2016-06-06 Thread Daniel Rocha
Alguém poderia, por favor, solucionar o problema abaixo: Ache a solução real da equação: log[(sqrt 2)^(x-2)] = x -- Esta mensagem foi verificada pelo sistema de antiv�rus e acredita-se estar livre de perigo.

[obm-l] Dúvida num Enunciado

2018-04-25 Thread Luiz Antonio Rodrigues
Olá, pessoal! Boa tarde! Estou tentando fazer o exercício abaixo, mas o problema é que eu não entendi o enunciado... Determine para quais valores de r (r>0) a implicação é verdadeira: |x+3| x^2 - 10x + 9 > 0 para todo x real. Agradeço qualquer ajuda! Um abraço! Luiz -- Esta mensagem foi verific

[obm-l] Enc: Re: DÚVIDA!

2004-12-09 Thread jorgeluis
- Mensagem encaminhada de Otto Nogami <[EMAIL PROTECTED]> - Data: Tue, 7 Dec 2004 23:20:38 -0200 De: Otto Nogami <[EMAIL PROTECTED]> Reponder para: Otto Nogami <[EMAIL PROTECTED]> Assunto: Re: DÚVIDA! Para: [EMAIL PROTECTED] Detalhe: se ele não tiver to

[obm-l] Enc: Re: DÚVIDA!

2004-12-09 Thread jorgeluis
- Mensagem encaminhada de Otto Nogami <[EMAIL PROTECTED]> - Data: Tue, 7 Dec 2004 23:19:38 -0200 De: Otto Nogami <[EMAIL PROTECTED]> Reponder para: Otto Nogami <[EMAIL PROTECTED]> Assunto: Re: DÚVIDA! Para: [EMAIL PROTECTED] Mais especificamen

Re: [obm-l] Dúvida - conjuntos

2005-07-28 Thread Daniel S. Braz
As imagens não estão aparecendo... Em 27/07/05, admath<[EMAIL PROTECTED]> escreveu: > A dúvida encontra-se em: > > http://www.admath.cjb.net > > Obrigado. > > > Yahoo! Acesso Grátis: Internet rápida e gr

Re: [obm-l] Dúvida - conjuntos

2005-07-28 Thread admath
Acho que agora está aparecendo.   http://www.admath.cjb.net   Obrigado.   "Daniel S. Braz" <[EMAIL PROTECTED]> escreveu: As imagens não estão aparecendo...Em 27/07/05, admath<[EMAIL PROTECTED]> escreveu:> A dúvida encontra-se em:> > http://www.admath.cjb.net> &g

[obm-l] Dúvida sobre matrizes

2005-07-31 Thread Marcos Martinelli
Ao estudar matrizes elementares me deparei com o seguinte teorema, que não estava demonstrado no livro. Peço ajuda de vocês para que me guiem na demonstração do mesmo. Teorema: Toda matriz é equivalente por linhas a uma matriz em forma canônica reduzida por linhas. Desde já, agradeço. ===

Re: [obm-l] Dúvida - Geometria

2005-08-15 Thread saulo nilson
2/(74,16-z)=2,22/78,17 Z=3,74 On 8/14/05, admath <[EMAIL PROTECTED]> wrote: > Está em: > > http://www.admath.cjb.net > > Obrigado. > > > Yahoo! Acesso Grátis: Internet rápida e grátis. Instale o discador agora! > > =

Re: [obm-l] Dúvida - Geometria

2005-08-15 Thread Susanna
eu acho que a relação seria 2/(74,16-z)=2,22/(78,17-z) z=37,705... On 8/15/05, saulo nilson <[EMAIL PROTECTED]> wrote: > 2/(74,16-z)=2,22/78,17 > Z=3,74 > > On 8/14/05, admath <[EMAIL PROTECTED]> wrote: > > Está em: > > > > http://www.admath.cjb.net > > > > Obrigado. > > > >

[obm-l] divida ou dúvida

2005-08-15 Thread Hermann
Caros companheiros da lista, me deem uma luz.   De quantos modos diferentes um produto com 3 fatores(pode ser repetido, tipo 36.1.1 ou 2.2.9)  dá 36 ?   Abraços Hermann    

[obm-l] Dúvida - Mat. básica

2005-09-04 Thread admath
olá!   Tenho a seguinte equação:   [(x-1)(x-3)] / x^3 + 2x - 3 = 0   Pq não posso passar x^3 + 2x - 3 multiplicando com 0 ? Em que casos posso fazer isso? Em que casos não posso?   Quanto a equação:   x(x-2) = x(x-3)   Pq posso cancelar o x? Há algum caso que eu não possa fazer isso?   São dúvidas

Re: [obm-l] dúvida - inequecão

2005-09-20 Thread Eduardo Wilner
Tem certeza que e x^2+mx-2/x^2-x+1 ? Nao falta parenteses? Poderia-se agrupar (m-1)x como nx . --- "Emanuel Carlos de A. Valente" <[EMAIL PROTECTED]> escreveu: > salve lista meu gabarito não está batendo, me > acudam: > -3 gab: -1 meu result: m<-7 ou m>2 > > > ===

Re: [obm-l] dúvida - inequecão

2005-09-21 Thread Danilo Nascimento
Ola   -3<(x^2+mx-2)/(x^2-x+1)<2 Tomando a primeira desigualdade : -3<(x^2+mx-2)/(x^2-x+1)(para agilizar multiplique cruzado já q o denominador é sempre positivo para qualquer valor de x) desenvolva e chegue em 4x^2 +x(m-3) + 1>0 como vc quer q o trinomio seja sempre positivo para qualquer valor de

Re: [obm-l] DÚVIDA CRUEL

2005-10-31 Thread Nicolau C. Saldanha
On Sun, Oct 30, 2005 at 08:58:51PM +, Robÿe9rio Alves wrote: > e POSSÍVEL ter uma reta com infinitos pontos, mais 1 ponto ? Caso Seja > responda matematicamente > > e POSSÍVEL ter uma reta com infinitos pontos mais 1 ponto ? Caso Seja > responda matematicamente A sua pergunta é muito

Re: [obm-l] DÚVIDA CRUEL

2005-10-31 Thread Chicao Valadares
uma PG infinita convergente + 1 eh possivel, agora uma reta, talvez seja uma questao de Analise , coisa que eu infelizmente nunca tive a oportunidade de estudar.. --- Robÿe9rio Alves <[EMAIL PROTECTED]> escreveu: > e POSSÍVEL ter uma reta com infinitos pontos, mais > 1 ponto ? Caso Seja r

Re: [obm-l] Dúvida Raiz

2005-12-31 Thread Luiz Miletto
sqrt(4)=2  significa: Qual o número que multiplicado por ele mesmo resulta 4? (+2)x(+2)=4 (-2)*(-2)=4 Portanto as duas raízes, (-2) e (+2), satizfazem a pergunta. sqrt(4)=+-2 está correto.  Em 30/12/05, Bruna Carvalho <[EMAIL PROTECTED]> escreveu: Tô com uma dúvida bem simples de algo que

Re: [obm-l] Dúvida Raiz

2005-12-31 Thread Bruna Carvalho
Então pq usamos somente a raiz positiva??

[obm-l] Dúvida em Lógica

2006-02-21 Thread Rhilbert Rivera
Amigos, só estou fazendo essa pergunta porque não encontrei nos livros que tenho a resposta. Numa proposição que tenha simultaneamente o "ou" ( V) e o "ou exclusivo" (|V|) o que devo fazer primeiro? E por quê? Por exemplo:  " p V q |V| r ". E se tivermos  simultaneamente "ou" (V) e o "e"(^)? Por e

[obm-l] dúvida sobre notação

2006-03-21 Thread cleber vieira
Amigos por favor me respondam.Quando você escreve que os triângulos  ABC e KLB são semelhantes faz alguma diferença em colocar ABC e BKL(semelhantes), desde que você respeite a razão de semelhança ?   Um professor pediu para que eu resolvesse um exercício no quadro e quando coloquei ABC e KLB

[obm-l] Dúvida conceitual (equações)

2018-10-14 Thread Vanderlei Nemitz
Bom dia! Na seguinte questão, que me foi apresentada por um aluno, a resposta proposta é a alternativa C (1/2). Eu sempre pensei que apenas considerávamos multiplicidades em equações polinomiais. Como essa é uma equação exponencial, obtive a resposta B (-1/2). O que é correto pensar? O produto das

[obm-l] Dúvida e ajuda.

2022-04-08 Thread Pedro José
Bom dia! Posso concluir que um número representado por uma infinidade de algarismos decimais é racional se e somente se tem um período de repetições desses algarismos? A ida é fácil se tiver o período é racional. Já a volta não sei se é verdade e se for há como provar? Meu objetivo primário é sabe

Dúvida sobre exame do ITA

2001-10-04 Thread Alex Vieira
Saudacoes a todos os colegas da lista,   Irei prestar ITA este ano, e, apos resolver algumas provas dos anos anteriores, verifiquei que nas materias exatas ocorrem diversas questoes complicadissimas, senao polemicas, com assuntos que "atrapalham" ateh professores.   Como sei que nesta lista d

Re: DÚVIDA DE CÁLCULO I

2001-10-31 Thread bmat
Olha, eu acho que tem duas saídas, uma que eu chamaria de rápida e outra que seria mais "normal" para atacar o problema logo de cara e com certeza chegar à resposta. Vamos lá, primeiro pela maneira normal: Ache os pontos de intersecção da reta com a parábola resolvendo o sistema y=x^2 e y=5x+11.

Re: DÚVIDA DE CÁLCULO I

2001-11-01 Thread Jose Paulo Carneiro
). JP - Original Message - From: <[EMAIL PROTECTED]> To: <[EMAIL PROTECTED]> Sent: Wednesday, October 31, 2001 11:31 PM Subject: Re: DÚVIDA DE CÁLCULO I Olha, eu acho que tem duas saídas, uma que eu chamaria de rápida e outra que seria mais "normal" para atacar o probl

Re: [obm-l] trigonometria dúvida

2002-02-14 Thread Marcelo Souza
ou. Se vc naum conseguir provar eu posso mandar a prova pra vc por email. ok Abracos Marcelo >From: "haroldo" <[EMAIL PROTECTED]> >Reply-To: [EMAIL PROTECTED] >To: <[EMAIL PROTECTED]> >Subject: [obm-l] trigonometria dúvida >Date: Thu, 14 Feb 2002 00:01:15

[obm-l] ALGEBRA LINEAR: dúvida

2002-04-01 Thread .SamueL.
nte dependente, então q>=p (II) Se A é linearmente independente e q>=n então q>=p (III) Se A e B são linearmente independentes então p<=n e q<=n   A resposta diz que a II e a III estão corretas. Minha dúvida é a seguinte:   Porque a I está errada? Seria porque não sabemos se A

Re: [obm-l] Dúvida - Estatística

2002-04-16 Thread Augusto César Morgado
a) 104 com prob 0,4^4 (4 subidas) 102 com prob C(4,3) * (0,4^3) * 0,6 (3 subidas e 1 descida) 100 com prob C(4,2) * (0,4^2) * (0,6*2) (2 subidas e 2 descidas) 98 com prob C(4,1) * (0,4^1) * (0,6*3) (1 subida e 3 descidas) 96 com prob 0,6*4 (4 descidas) b) O aumento médio por dia é 1*0,4-1*0,6=

[obm-l] Dúvida de Combinatoria

2002-07-31 Thread leonardo mattos
Tendo n-casais(marido e mulher)de quantas maneiras diferentes pode-se formar n/2 grupos de tal forma q em que cada grupo contenha 2 pessoas,ou seja,não importa a ordem.Uma maneira seria [(H1,M1),(H2,M2),(H3,M3)...,(Hn,Mn)]. O que é permutação caótica?

Re: [obm-l] dúvida polinômios

2002-11-30 Thread joao dias
At 18:06 29/11/2002 -0200, you wrote: Olá companheiros de lista, Eu gostaria de saber o que há de especial quando temos p(x) = p(x-k) ou p(x)=p(k-x),k real,para um polinômio qualquer.Tipo,que informações interessantes podem ser retiradas de uma igualdade dessas,como essas igualdades podem ser

Re: [obm-l] dúvida combinatória

2002-12-06 Thread Augusto César Morgado
Chamemos as pessoas de A, B, C. Supondo os 5 livros diferentes, ha 3^5 = 243 modos de distribui-los (o primeiro livro pode ser distribuído de 3 modos, o segundo de 3 modos etc). Ha 2^5 = 32 modos de distribui-los apenas a A e B, 32 a A e C etc. Ha 1 modo de distribui-los apens a A, 1 a B etc. A r

[obm-l] Uma dúvida simples

2002-12-18 Thread Felipe Villela Dias
Pessoal, desculpem o meu nível de ignorância mas me surgiu uma dúvida: todos os números primos possuem raízes quadradas irracionais? Caso positivo, existe alguma prova simples para isso?   ---Outgoing mail is certified Virus Free.Checked by AVG anti-virus system (http://www.grisoft.com

[obm-l] Dúvida Da Fuvest

2003-01-11 Thread Helder Oliveira de Castro
Olá pessoal da lista,   A minha dúvida é quanto a primeira questão da Fuvest de Matemática da 2ª Fase (parte b). Aí vai: (b) Quantos múltiplos de 9 ou 15 há entre 100 e 1000? [Notações] Sejam: p: o nº de múltiplos de 9 entre 100 e 1000 q: o nº de múltiplos de 15 entre 100 e 1000 r: o nº de

[obm-l] dúvida sobre relações

2004-02-29 Thread kleinad
Recentemente, postaram algo sobre relações entre conjuntos, e eu fiquei com uma dúvida. Todas as relações de A em B se fazem associando-se 1 subconjunto de A a 1 subconjunto de B, exceto o vazio? Por exemplo, para A = { 1, 2, 3 } e B = { 4, 5, 6 }, ({ 1, 2 } , {4}) é uma relação de A em B ? E

Re: RES: [obm-l] dúvida

2004-03-26 Thread ariel
e escrevendo no papel: Tg[Pi/3] = h / x Tg[Pi/6] = h / (x+d) sqrt(3) = h / x ==> x = h/sqrt(3) sqrt(3)/3 = h / (x+d) sqrt(3)/3 = h / ((h/sqrt(3)+d) h + d.sqrt(3) = 3h d = 2h/sqrt(3) d = 2h.sqrt(3)/3 bom, o papel retorna o mesmo resultado.. rs David M. Cardoso escreveu: Colocando esse

Re: RES: [obm-l] dúvida

2004-03-27 Thread Faelccmm
Esclarecam-me uma duvida: Suponho que h seja a altura de um eixo imaginario (perpendicular ao solo) que vai do solo ateh a bala, passando pelo corpo da ave, certo ? Logo h eh crescente no intervalo [0,90º[, ou seja, a inclinacao da arma produz um angulo alfa, cujo cateto oposto eh h, certo ? Entao

Re: RES: [obm-l] dúvida

2004-03-27 Thread kleinad
Me chama atenção que não está sendo considerado o fato do homem não estar com a arma nos pés... ariel ([EMAIL PROTECTED]) escreveu: > >e escrevendo no papel: > >Tg[Pi/3] = h / x >Tg[Pi/6] = h / (x+d) > >sqrt(3) = h / x ==> x = h/sqrt(3) >sqrt(3)/3 = h / (x+d) > >sqrt(3)/3 = h / ((h/sqrt(3)+d) >h

[obm-l] dúvida de limites

2004-04-07 Thread André Martin Timpanaro
Quando vale que: lim (x---> a) f(g(x)) = lim (x--->lim(x---> a) g(x)) f(x) ? André T. _ MSN Messenger: converse com os seus amigos online. http://messenger.msn.com.br ==

Re: [obm-l] Dúvida persistente!!!

2004-04-10 Thread Faelccmm
A area procurada seria +/- esta, em amarelo, que estou enviando em *gif ? Em uma mensagem de 10/4/2004 19:07:14 Hora padrão leste da Am. Sul, [EMAIL PROTECTED] escreveu: Olá pessoal, Tenho uma dúvida que já perdura por anos. Gostaria de compartilhar com vocês, e se a resposta já foi

RE: [obm-l] Dúvida persistente!!!

2004-04-10 Thread Qwert Smith
]> Reply-To: [EMAIL PROTECTED] To: [EMAIL PROTECTED] Subject: [obm-l] Dúvida persistente!!! Date: Sat, 10 Apr 2004 19:02:35 -0300 Olá pessoal, Tenho uma dúvida que já perdura por anos. Gostaria de compartilhar com vocês, e se a resposta já foi lançada na lista, gostaria apenas que indicassem o cami

[obm-l] Dúvida em identidade

2004-04-16 Thread Maurizio
Estou com dificuldade nesse problema: Se G é o baricentro de ABC e P(x,y) é um ponto qualquer, prove que: PA^2+PB^2+PC^2=3PG^2+AG^2+BG^2+CG^2 Se alguém conseguir resolver ficaria muito grato. Obrigado, MauZ = Instruções pa

[obm-l] Dúvida de Conceito

2004-04-20 Thread Gustavo
A função afim :  1) tem como gráfico uma qualquer reta,uma reta não vertical  ou uma reta obrigatóriamente inclinada (oblíqua )em relação aos eixos?. 2) é sinonimo de função polinomial do primeiro grau?   3) é uma função do tipo f(x) = ax+b ,com a e b reais e  a diferente de zero , ou a pod

Re: [obm-l] dúvida(física)

2004-04-24 Thread Wellington Assis
Pense na conservação da quantidade de movimento...   TSD <[EMAIL PROTECTED]> wrote: A pressão interna numa garrafa de refrigerante fez, emdeterminado momento, com que a rolha que tampava agarrafa fosse lançada violentamente para longe dagarrafa. Se a massa da garrafa é 99 vezes maior do que ama

[obm-l] Dúvida de Análise!!!

2004-06-05 Thread Lista OBM
Gostaria que alguém me ajudasse com o problema abaixo:   Seja f:R --> R contínua, com lim{x->+inf} f(x) = + infinito e lim{x->-inf} f(x) = - infinito. Prove que, para todo c em R dadoo, existe entre as raízes x da equação f(x) = c uma cujo módulo |x| é mínimo.   Obs.: Não estou conseguindo interpre

[obm-l] dúvida em combinatória

2004-06-05 Thread TSD
de quantas ,maneiras um estudande poderá entrar e sair de 8 portas. poderia explicad por favor!!!

[obm-l] Dúvida sobre álgebra

2004-06-18 Thread João Paulo
Prezados amigos da lista, eu gostaria de saber porque o seguinte fato (aparentemente óbvio), mas que eu não consegui argumentos, é verdade: Z[t]/(t^2 - 2,3t -1) não é isomorfo à Z[sqrt(2),1/3]. desde já agradeço, []'s João. -- ___ Sign-up f

[obm-l] Dúvida de limite

2004-07-16 Thread Renan de Oliveira e Silva
Oi!Sou novo na lista(na verdade,sou leitor há tempo e só agora resolvi participar!) Tenho uma dúvida simples sobre um limite q não consigo resolver sem usar as regras de L'Hospittal.Parece q ele foi questão de uma prova da Escola Naval. Eis: lim{[1/2(1-x)^1/2]-[1/3(1-x)^1/3]} x->1 Eu a

[obm-l] Dúvida em demonstração

2003-02-25 Thread Henrique Branco
Hi all!   Discutindo com um amigo meu sobre a demonstração das propriedades do logaritmo natural, encontrada no "Calculo com Geometria Analítica", do Swokowsky, ele argumentou que a mesma seria falha. Vou expor a prova encontrada no livro citado e depois discutir.   Propriedade: Se p > 0 e q

[obm-l] Dúvida de vestibular

2003-02-26 Thread okakamo kokobongo
Oi Pessoal, Estava estudando análise combinatória por uma apostila de um curso pré-vestibular, e encontrei o seguinte problema, que achei interessante, mas minha solução foi muito longa, e não sei se está certa, porque tinha muitos casos. Se estivesse num vestibular, o que faria? Num país,

[obm-l] dúvida álgebra linear

2003-03-28 Thread rafaelc.l
Minha dúvida é bem básica, referente a seguinte afirmação: - Dois vetores quaiquer são sempre coplanares. Mas podemos imaginar dois segmentos orientados não coplanares.Só que a teoria diz tbm que os vetores representados por tais segmentos orientados são coplanares pois podemos tomar

Re: [obm-l] Dúvida-Geo.Esp.

2003-06-30 Thread A. C. Morgado
Voce nao devia estar de acordo era com o enunciado que estah horrendo. A resposta pode ser 1: nao ha nada no enunciado que impeça as duas retas de serem coplanares. Pode ser tambem 5: os 3 planos do gabarito e outros 2 planos determinados pela reta dos 3 pontos com cada um dos dois pontos fora

[obm-l] estou com dúvida...

2003-07-20 Thread elton francisco ferreira
O número de raízes reais distintas da equação x|x|-3x + 2=0 é? a)0 b)1 c)2 d)3 e)4 ___ Yahoo! Mail Mais espaço, mais segurança e gratuito: caixa postal de 6MB, antivírus, proteção contra spam. http://br.mail.yahoo.com/ =

Re: [obm-l] DÚVIDA...correçao

2003-08-14 Thread Augusto Cesar de Oliveira Morgado
Eh, eh, eh! Nossa, fiz 3 ao quadrado igual a 27! Estah corrigido abaixo. Em Thu, 14 Aug 2003 20:58:08 -0300, "A. C. Morgado" <[EMAIL PROTECTED]> disse: Henrique Patrício Sant'Anna Branco wrote: Não tenho muita certeza das minha resoluções, mas já que ninguém respondeu,vai aí alguma tentativa.

Re: [obm-l] Dúvida - Combinatória

2003-08-15 Thread Claudio Buffara
Title: Re: [obm-l] Dúvida - Combinatória on 15.08.03 08:17, Celso Junior dos Santos Francisco at [EMAIL PROTECTED] wrote: Sobre uma reta r e uma outra paralela a ela , marcam-se 13 , sendo que a maioria deles sobre r . Sabendo que a razão entre o número de quadriláteros e o número de

Re: [obm-l] dúvida Epcar

2003-09-02 Thread Claudio Buffara
on 02.09.03 01:02, [EMAIL PROTECTED] at [EMAIL PROTECTED] wrote: > alguém poderia dar uma ajudinha. por favor nesta questão; > > De quantos modos 3 casais podem sentar-se ao redor de uma mesa circular > de tal forma que marido e mulher não fiquem juntos? > > r= 32. como faço isso ?? > Eu

[obm-l] dúvida nums problemas.

2003-09-27 Thread tarciosd
representar no plano de argand-Gaus, as imagens das raízes quartas de um número complexo z, não nulo, são vértices de um quadrado inscrito em uma circunferência de centro na origem do plano e cujo raio é igual a |z|. Na figura seguinte, o ponto A representa uma das imagens das raízes quartas de

Re: [obm-l] Dúvida-áreas

2003-10-02 Thread Claudio Buffara
Title: Re: [obm-l] Dúvida-áreas on 01.10.03 23:44, Fábio Bernardo at [EMAIL PROTECTED] wrote: Oi pessoal, ajudem-me por favor. Não estou conseguindo visualizar a área pedida e portanto, não consigo achar a resposta. Desde já agradeço. Para proteger um terreno circular com raio de 12m, amarra

Re: [obm-l] dúvida 2

2003-12-02 Thread Johann Peter Gustav Lejeune Dirichlet
Nao faço muita ideia mas acho que usa o fato de que o angulo central e o angulo externo de um 20-agono e de dezoito graus.Isto tem a ver com o problema que Gauss resolveu com Galois Talvez usando trigonometria saia...Tente assim:faça um desenho e calcule o lado de um 20-agono de raio 1.Talvez seja

Re: [obm-l] dúvida 2

2003-12-03 Thread Rafael
Peter, Você que gosta, uma vez eu achei esse valor usando trigonometria assim: Ache o cos 5x em função do cos x cos 5x = (cos 2x).(cos 3x) - (sen 3x).(sen 2x) E quando chegar numa expressão só em função de cos x você iguala cos 5x = cos 90° Não foi tão difícil... Abraços, Rafael. --- Johann

Re: [obm-l] dúvida/grupos

2003-12-03 Thread Claudio Buffara
on 03.12.03 18:01, [EMAIL PROTECTED] at [EMAIL PROTECTED] wrote: > Estou com a seguinte dúvida: > > Seja G um grupo finito de elemento neutro e. Então > > 1)qualquer a pertencente a G, ordem (a) divide |G |. > > 2) qualq. a pert. a G, a^|G|=e > > 3)qualq. a pert. G

Re: [obm-l] dúvida/grupos

2003-12-03 Thread Claudio Buffara
on 03.12.03 19:00, Claudio Buffara at [EMAIL PROTECTED] wrote: > on 03.12.03 18:01, [EMAIL PROTECTED] at [EMAIL PROTECTED] wrote: >> >> Seja G um grupo finito de elemento neutro e. Então >> >> 3)qualq. a pert. G, a é diferente de e, a^m=e=> |G| | m. >> > a^m = e ==> m | |G|. E isso eh outra con

Re: [obm-l] dúvida 2

2003-12-04 Thread Johann Peter Gustav Lejeune Dirichlet
Verdade...E bem mais fazcil do que eu imaginava... Mas a ideia e de que quase sempre e possivel ver um poligono regular...E eu estava tentando uma resposta cearenseRafael <[EMAIL PROTECTED]> wrote: Peter,Você que gosta, uma vez eu achei esse valor usandotrigonometria assim:Ache o cos 5x em função d

Re: [obm-l] dúvida 2

2003-12-04 Thread Johann Peter Gustav Lejeune Dirichlet
Ah,ja contei como o Gauss construiu o poligono de 17 lados?Johann Peter Gustav Lejeune Dirichlet <[EMAIL PROTECTED]> wrote: Verdade...E bem mais fazcil do que eu imaginava... Mas a ideia e de que quase sempre e possivel ver um poligono regular...E eu estava tentando uma resposta cearenseRafael <[E

Re: [obm-l] dúvida 2

2003-12-05 Thread Rafael
--- Johann Peter Gustav Lejeune Dirichlet <[EMAIL PROTECTED]> escreveu: > Ah,ja contei como o Gauss construiu o poligono de 17 > lados? Não. Por favor, satisfaça minha curiosidade. Rafael. __ Yahoo! Mail: 6MB, anti-spam e anti

Re: [obm-l] dúvida 2

2003-12-05 Thread Rafael
Achei a minha resolução: Sabendo que: cos 3x = 4cos³ x - 3cos x sen 3x = 3sen x - 4sen³ x Podemos fazer: = cos 5x = cos (3x + 2x) = (cos 3x).(cos 2x) - (sen 3x).(sen 2x) = (4cos³ x - 3cos x).(cos² x - sen² x) - (3sen x - 4sen³ x).[2.(sen x).(cos x)] = 4.(cos x)^5 - 4.(cos³ x).(sen² x) - 3cos³ x +

Re: [obm-l] dúvida 2

2003-12-05 Thread Johann Peter Gustav Lejeune Dirichlet
Bem,quem ta a fim de calcular cos pi/17?Rafael <[EMAIL PROTECTED]> wrote: Achei a minha resolução:Sabendo que:cos 3x = 4cos³ x - 3cos xsen 3x = 3sen x - 4sen³ xPodemos fazer:= cos 5x= cos (3x + 2x)= (cos 3x).(cos 2x) - (sen 3x).(sen 2x)= (4cos³ x - 3cos x).(cos² x - sen² x) - (3sen x -4sen³ x).[2.(

Re: [obm-l] dúvida 2

2003-12-05 Thread Ricardo Bittencourt
Johann Peter Gustav Lejeune Dirichlet wrote: Bem,quem ta a fim de calcular cos pi/17? Aqui tem o seno de pi/17, pra tirar o cos é só fazer sqrt(1-sin*sin): http://www.jimloy.com/geometry/17-gon0.gif Eu queria mesmo é ver sen 1 por extenso... Só precisa fazer cos3=cos18-cos15 e depois

Re: [obm-l] dúvida 2

2003-12-05 Thread Guilherme Carlos Moreira e Silva
legal! mas a parte que você usa o angulo de 15° . . não há outro jeito de escolhermos um dos dois?Rafael <[EMAIL PROTECTED]> wrote: Achei a minha resolução:Sabendo que:cos 3x = 4cos³ x - 3cos xsen 3x = 3sen x - 4sen³ xPodemos fazer:= cos 5x= cos (3x + 2x)= (cos 3x).(cos 2x) - (sen 3x).(sen 2x)= (4c

Re: [obm-l] OUTRA DÚVIDA

2003-12-13 Thread Ariel de Silvio
da formula tgx=|(mr-ms)/(1+mr*ms)| sendo mr => coeficiente angular de r ms => coeficiente angular de s tgx => angulo formado entre as duas retas tgx=|5/-5| tgx=1 .:. x=45 *** MENSAGEM ORIGINAL *** As 16:50 de 13/12/2003 [EMAIL PROTECTED] escreveu: >PODERIAM AJUDA NESTA OUTRA

Re:_[obm-l]_característica_de_um_corpo/dúvida

2004-01-07 Thread Carlos Maçaranduba
Porque a conclusão de que é infinito???Essa prova não mostra somente que se a caracteristica é zero, os elementos são distintos 2 a 2 no corpo --- Felipe Pina <[EMAIL PROTECTED]> escreveu: > Vou resolver a 1) > > olhe para os seguintes elementos do corpo > > 1 > 1 + 1 > 1 + 1 + 1 > 1 + 1 +

Re: [obm-l] Dúvida Simples!!!

2004-01-13 Thread Jefferson Franca
Que tal se vc usar a definição? Um abraçoCarlos Alberto <[EMAIL PROTECTED]> wrote: Existe alguma maneira analitica, ou prática. Para que eu possa provar se uma função é sobrejetora, injetora e bijetora. Por exemplo: F: R -> R tal que f(x) = 2x-5 g: R-{4} -> R-{1} tal que g(x) = x+1/x-4 Eu só consi

Re: [obm-l] 1o Dúvida

2004-01-27 Thread Villard
; ), temos n - n' =(n-S(n))-(n'-S(n')) que é a diferença de dois múltiplos de 9, o que dá um múltiplo de 9.Abraços,  Villard - Mensagem Original De: [EMAIL PROTECTED]Para: "[EMAIL PROTECTED]" <[EMAIL PROTECTED]>Assunto: [obm-l] 1o DúvidaData: 27/01/04

Dúvida referente a um exercício

2000-02-11 Thread Marcos Eike Tinen dos Santos @ ITA @
Em volta de uma mesa redonda estão sentados representantes de n países ( n> =2), satisfazendo a seguinte condição: Se duas pessoas são do mesmo país, então, seus respectivos vizinhos da direita não podem ser de um mesmo país. Determine, para cada n, o número máximo de pessoas que pode haver em vol

Geometria- dúvida na minha demonstração

2000-05-03 Thread Marcos Eike Tinen dos Santos
olá pessoal, Venho pedir-lhe uma demostração para este problema, eu fiz aqui, só que tenho dúvidas. ABCD um quadrilátero convexo, e O a intersecção de suas diagonais, seja L,M,N os pontos médios de DB,BC e CA, suponha que AL, OM e DN seja concorrentes. Mostre que AD || BC. Ats, Marcos Eike

Re: Re: Dúvida sobre Resíduos

2000-11-08 Thread alexv
Oi Marcos , Gostaria de fazer um acréscimo ao que o JP já sugeriu: Se não me engano (eu não tenho o livro neste momento em mãos) é possível encontrar conteúdo sobre o assunto em: Teoria da Congruências (Edgard de Alencar Filho). É um bom livro mas acredito estar esgotado (Eu acredito, mas posso

<    1   2   3   4   5   6   7   8   9   10   >